User Avatar
hayworthzm452
Joined
Apr 2025
Subscription
Free

Hi community,

Looking to form a group of motivated studiers to BR one PT/week over the summer.

Anyone who wants to join can do so using this link -- no need to message me directly or post your email!

https://groupme.com/join_group/59882617/BjYFugwa

I've learned from participating in the community here that talking through problems is the best way for me to learn at this stage of my studying. I'm taking the July LSAT and it's my third and last chance to get a 174+. I've been scoring mid 170s on PTs, and I want to get serious with a group, because I know that's the only way to make sure my performance on test day is bulletproof.

All levels are welcome!

I find it best when there is a mix!

Looking forward to studying!

Zach

User Avatar
hayworthzm452
Wednesday, May 20 2020

Hi! Can you also add me? Also looking to study seriously.

Email: hayworthzm@.harvard.edu

User Avatar
hayworthzm452
Tuesday, May 19 2020

Please add me if it's not too late: hayworthzm@.harvard.edu

Thanks!

User Avatar
hayworthzm452
Tuesday, May 19 2020

I just finished my LSAT Flex. I had no technical issues at all and really enjoyed taking it at home.

Similar to what others have said, LR seemed totally normal, LG on the easier side, RC on the harder side (really just quite long). Looking forward to June 5!

Congratulations to everyone who completed the test.

PrepTests ·
PT133.S2.Q20
User Avatar
hayworthzm452
Tuesday, May 12 2020

This question really stumped me, both the first and second time I looked at it (the first time I got it wrong I put it in my "tough questions" log to come back to in the future). Once I figured it out, I learned an important lesson about a nuance of sufficient assumptions.

The stimulus states that J was murdered, and if a person murdered him, that person was necessarily in J's office on the day of the murder (Murderer → in office). We learn that S and H were both in the office, so they satisfy the necessary condition for being the murderer. We then learn that if H had done the murder, he would have left footprints or fingerprints behind, and if S were the murderer, she would leave neither footprints nor fingerprints behind. So HM → Footprints or Fingerprints; SM → Footprints and SM → Fingerprints. Only Fingerprints were found, but they weren't H's. Therefore: S was the murderer.

Analysis: Immediately it occurred to me that there were two problems with this argument. The first is that the argument requires the assumption that only H and S were in the office on that day. The second is that the argument seems to require the assumption that the fingerprints were not Samantha's. At first read, I thought that both were needed in order to make the argument valid. Thus I was confused by the stem, which asks for a single sufficient assumption.

In reality, though, the two necessary assumptions are not equally powerful. Let's start with what we know: The only prints in the room did not belong to H therefore we have a negated contrapositive of (HM → Footprints or Fingerprints) and hence H is definitely not the murderer. Now if we grant the assumption that the fingerprints were not Samanthas, we are a micro-step closer to a valid argument, but all we have done is satisfy the necessary condition of her being a murderer, we haven't provided a sufficient condition. We still need to guarantee that no one else was in the room and that Samantha is our only option for murderer in order to have a valid argument. However, if we go the other way and prove off the bat that there was no one else int he room besides H and S, we have no other option besides S to implicate as murderer, since we have already validly exculpated H.

The answer choices:

A) This is like the answer choice E about Samantha's fingerprints in that it is another necessary assumption required by the argument. But it is also automatically established if the condition about S and H being the only ones in the office is fulfilled.

B) Not relevant

C) Bingo

D) The argument states this explicitly, it is also not a sufficient assumption

E) a necessary, not a sufficient assumption

Hi community,

Two of us are planning to meet via Zoom this evening to start BR-ing PT 79. Looking for joiners. If you want to join, here is the link to a GroupMe group where I will post the Zoom link closer to the target time (9:30 PM EST):

https://groupme.com/join_group/59756531/VhuncmF5

If you don't have GroupMe, DM me here and I will send you the link that way. Maybe we will do more PTs after 79.

Happy studying!

Zach

User Avatar
hayworthzm452
Saturday, May 09 2020

I'm aiming for July, but would love to be on board with you guys. I will DM op as well.

User Avatar
hayworthzm452
Saturday, Jun 06 2020

@ said:

Anyone surprised at their score? Acc to mine I got a 166 which is the equivalent of 17 wrong, I was a bit shocked at this. I was thinking if you basically double the # wrong in LR you'd get a pretty close estimate of a non-flex test score, I have no idea what's going on with how flex is graded. Would love to see at least a score breakdown

My score was almost exactly my PT average. Was yours above or below?

User Avatar

Friday, Jun 05 2020

hayworthzm452

Thank you 7Sage Community! [164 to 174]

Just opened my score from the May Flex and I wanted to come straight here and extend a huge THANK YOU to the community. I could not have done this without all of you. I have grown so much through this process. Respect the grind!

Confirm action

Are you sure?